In a box of nerds candy, the ratio of pink to purple candies is 19:20. if there are 429 pieces of candy in the box, how many are pink?

Answers

Answer 1

There are 199 pink candies in the box of Nerds calculated on the basis of given information.

To find out, you first need to add the ratio of pink and purple candies, which is 19+20=39. Then, divide the total number of candies by the sum of the ratio to find the value of one unit of the ratio, which is 429/39 = 11.

Then, multiply the value of one unit of the ratio by the value of the pink candies, which is 19, to find the number of pink candies, which is 11 x 19 = 209. Therefore, there are 209 purple candies in the box.

For more questions like Candies click the link below:

https://brainly.com/question/22074890

#SPJ11


Related Questions

Rule: y is 8 less than 4 times x

Answers

Answer:

y = 8 - 4x

" is" is express the equal sighn

"less than" express - sighn so you will write the equestion as

y = 8 - 4x

f(x, y) = ex sin y do first and second order partial derivatives. f(x, y) = e^x sin y. do first and second order partial derivatives

Answers

The first order partial derivatives of f(x, y) =  ex cos y and the second order partial derivatives are ex sin y = ex cos y.

The given function is f(x, y) = ex sin y. We need to find the first and second order partial derivatives of this function with respect to x and y.
First order partial derivatives:
To find the partial derivative of f(x, y) with respect to x, we treat y as a constant and differentiate ex with respect to x. This gives:
∂f/∂x = ex sin y
To find the partial derivative of f(x, y) with respect to y, we treat x as a constant and differentiate sin y with respect to y. This gives:
∂f/∂y = ex cos y
Second order partial derivatives:
To find the second order partial derivatives, we differentiate the first order partial derivatives we found above. That is, we differentiate ∂f/∂x and ∂f/∂y with respect to x and y, respectively.
∂/∂x (ex sin y) = ex sin y
∂/∂y (ex cos y) = -ex sin y
To find the mixed partial derivatives, we differentiate one of the first order partial derivatives with respect to the other variable. That is,
∂/∂y (ex sin y) = ex cos y
We can also find the mixed partial derivative  by differentiating ∂f/∂y with respect to x, which gives the same result:
∂/∂x (ex cos y) = ex cos y
The first order partial derivatives of f(x, y) = ex sin y are ∂f/∂x = ex sin y and ∂f/∂y = ex cos y and the second order partial derivatives are ex sin y, ∂f/∂[tex]y^2[/tex] = -ex sin y, and ∂f/∂x∂y = ∂f/∂y∂x = ex cos y.

For similar questions on partial derivatives:

https://brainly.com/question/30365299

#SPJ11

Allen opens a retirement savings account with an initial deposit of $5,000. he makes annual contributions to the account, and at the end of 5 years the account has grown to $8,650. which best describes allen’s investment?

a. allen invests in a retirement savings account that earns 5.5% interest compounded annually.

b. allen invests in a retirement savings account that earns 3% simple interest.

c. allen invests a retirement savings account that earns 2.75% interest compounded annually.

d. allen invests in a retirement savings account that earns 5.5% simple interest.

Answers

The answer would be

A = $8,650

P = $5,000

t = 5 years

Find out the compound interest?

To solve this problem, we can use the formula for compound interest:

A = P(1 + r/n)^(nt)

Where:

A = the amount of money after t years

P = the principal (initial deposit)

r = the annual interest rate

n = the number of times the interest is compounded per year

t = the number of years

We know that Allen opened a retirement savings account with an initial deposit of $5,000 and made annual contributions. After 5 years, the account grew to $8,650. We don't know the annual interest rate or how the interest is compounded, so we can use the formula to find out.

Let's assume that Allen made no additional contributions to the account after the initial deposit. Then:

A = $8,650

P = $5,000

t = 5 years

We can rearrange the formula to solve for r:

Learn more about Interest

https://brainly.com/question/29480777

#SPJ11

Help there is a picture of it

Answers

Answer:

5 19/20 miles

Imaginé que usted recibe semanalmente $50 y de esos destina el 10% de ahorros para comprar un pantalón que cuesta $40

Cuántas semanas tendrá que ahorrar?

Answers

It will take 8 weeks to save up for $40 pair of trousers.

How many weeks will it take to save for $40?

Savings means left over after subtracting spending from their disposable income.

We will calculate how money saved each week which is:

= 10% *  $50

=  $5.

So, this means we have $45 left after saving for a week.

To save up $40, we need to save for:

= 40/5

= 8 weeks.

Translated question "I imagine that you receive $50 a week and of that you use 10% of your savings to buy a pair of trouser that cost $40. How many weeks will you have to save?"

Read more about savings

brainly.com/question/25787382

#SPJ4

Hudson is designing a new board game, and is trying to figure out all the possible


outcomes. How many different possible outcomes are there if he spins a spinner with


three equal-sized sections labeled Walk, Run, Stop and spins a spinner with 5 equal-


sized sections labeled Monday, Tuesday, Wednesday, Thursday, Friday?

Answers

There are 15 different possible outcomes when Hudson spins both spinners in his board game.

To determine the total number of possible outcomes when Hudson spins both spinners, you need to multiply the number of outcomes on the first spinner (Walk, Run, Stop) by the number of outcomes on the second spinner (Monday, Tuesday, Wednesday, Thursday, Friday).

Step 1: Determine the number of outcomes on the first spinner. There are 3 outcomes: Walk, Run, and Stop.

Step 2: Determine the number of outcomes on the second spinner. There are 5 outcomes: Monday, Tuesday, Wednesday, Thursday, and Friday.

Step 3: Multiply the number of outcomes from both spinners. 3 outcomes on the first spinner multiplied by 5 outcomes on the second spinner equals 15 total possible outcomes.

So, there are 15 different possible outcomes when Hudson spins both spinners in his board game.

To know more about board game refer here:

https://brainly.com/question/13456434

#SPJ11

The value of y varies directly with x. which function represents the relationship between x and y if y = 18/5 when x = 24

Answers

The function that represents the relationship between x and y is y = 3/20 x

Since y varies directly with x, we can write the relationship between x and y as

y = kx

where k is the constant of proportionality.

y = 18/5 when x = 24

Substituting these values into the equation, we get:

18/5 = k(24)

Simplifying this equation, we get:

k = (18/5) / 24

k = (18/5 × 24)

k = 18/120

We can simplify this expression to:

k = 3/20

Therefore, the function that represents the relationship between x and y is y = 3/20 x

Learn more about proportionality here

https://brainly.com/question/29126727

#SPJ4

Use cylindrical coordinates to evaluate the triple integral ∫∫∫√(x^2 + y^2) dV where E is the solid bounded by the
circular paraboloid z = 9 - (x^2 + y^2) and the xy-plane.

Answers

The value of the triple integral ∫∫∫[tex]E \sqrt{(x^2 + y^2)} dV[/tex] over the solid bounded by the circular paraboloid [tex]z = 9 - (x^2 + y^2)[/tex] and the xy-plane is 486π/5.

To evaluate the triple integral ∫∫∫[tex]E \sqrt{(x^2 + y^2)} dV[/tex], where E is the solid bounded by the circular paraboloid [tex]z = 9 - (x^2 + y^2)[/tex] and the xy-plane, we can use cylindrical coordinates. In cylindrical coordinates, the equation of the paraboloid becomes:

[tex]z = 9 - (r^2)[/tex]

The limits of integration are:

0 ≤ r ≤ 3 (since the paraboloid intersects the xy-plane at z = 0 when r = 3)

0 ≤ θ ≤ 2π

0 ≤ z ≤ 9 - (r^2)

The triple integral becomes:

∫∫∫[tex]E √(x^2 + y^2) dV = ∫0^3 ∫0^2π ∫0^(9-r^2) r√(r^2) dz dθ dr[/tex]

Simplifying, we get:

∫∫∫[tex]E √(x^2 + y^2) dV = ∫0^3 ∫0^2π ∫0^(9-r^2) r^2 dz dθ dr[/tex]

Evaluating the innermost integral, we get:

∫[tex]0^(9-r^2) r^2 dz = (9-r^2)r^2[/tex]

Substituting this back into the triple integral, we get:

∫∫∫[tex]E √(x^2 + y^2) dV = ∫0^3 ∫0^2π (9-r^2)r^2 dθ dr[/tex]

Evaluating the remaining integrals, we get:

∫∫∫[tex]E √(x^2 + y^2) dV = ∫0^3 (9r^2 - r^4) dθ[/tex]

= 2π [243/5]

= 486π/5

Therefore, the value of the triple integral ∫∫∫[tex]E \sqrt{(x^2 + y^2)} dV[/tex] dV over the solid bounded by the circular paraboloid [tex]z = 9 - (x^2 + y^2)[/tex] and the xy-plane is 486π/5.

For more such questions on paraboloid visit:

https://brainly.com/question/17461465

#SPJ11

Use the normal approximation to find the indicated probability. the sample size is n, the population proportion of successes is p, and x is the number of successes in the sample.
n = 81, p = 0.5: p(x ≥ 46)
group of answer choices

0.1210

0.1335

0.8790

0.1446

Answers

We know that the indicated probability is approximately 0.1210.

To use the normal approximation, we need to check if the conditions for a normal approximation are met. In this case, we have:

np = 81 * 0.5 = 40.5 ≥ 10
n(1-p) = 81 * 0.5 = 40.5 ≥ 10

Since both conditions are met, we can use the normal approximation to find the probability.

First, we need to find the mean and standard deviation of the sampling distribution of sample proportions:

mean = np = 81 * 0.5 = 40.5
standard deviation = sqrt(np(1-p)) = sqrt(81 * 0.5 * 0.5) = 4.5

Next, we need to standardize the value of x:

z = (x - mean) / standard deviation
z = (46 - 40.5) / 4.5 = 1.22

Finally, we can use a standard normal table or calculator to find the probability:

P(z ≥ 1.22) = 0.1118

Therefore, the answer is approximately 0.1210.

To know more about probability refer here

https://brainly.com/question/11234923#

#SPJ11

Stat 2300 a survey was conducted that asked randomly selected students nationwide if they traveled outside the country for spring break. a 99% confidence interval for the proportion of all students who traveled outside the country is (0.0994, .1406). how many students were surveyed

Answers

The survey randomly selected students nationwide if they traveled outside the country for spring break and a 99% confidence interval which is 753 students.

The formula for a confidence interval for a proportion is:

CI = p ± z*sqrt((p*(1-p))/n)

where p is the sample proportion, n is the sample size, and z is the critical value from the standard normal distribution corresponding to the desired confidence level.

In this case, the confidence interval is given as (0.0994, 0.1406), which means:

p = (0.0994 + 0.1406) / 2 = 0.1200

The critical value for a 99% confidence interval is z = 2.576.

Substituting these values into the formula and solving for n, we get:

0.0206 = 2.576*sqrt((0.12*(1-0.12))/n)

Squaring both sides and solving for n, we get:

n = (2.576² * 0.12 * 0.88) / (0.0206²) = 752.3

Rounding up to the nearest integer, we get:

n = 753

Therefore, the survey included 753 students.

Know more about confidence interval here:

https://brainly.com/question/29680703

#SPJ11

In the shown figure, DE←→
is parallel to side BC¯¯¯¯¯¯¯¯
in triangle ABC
. If m∠B=52
°, what is m∠DAB
?


m∠DAB
=
°

Answers

Answer:

In triangle ABC, m∠BAC = 50°. If m∠ACB = 30°, then the triangle is triangle. If m∠ABC = 40°, then the triangle is triangle. If triangle ABC is isosceles, and AB = 6 and BC = 4, then AC =

Answer:

52 degrees

Step-by-step explanation: because i looked and they looked the same so i put 52 and it was right

What do you think causes the percent of filers to jump so dramatically between the under-18 group and the 18-26 group?

Answers

The significant increase in the percent of filers between the under-18 group and the 18-26 group can be attributed to various factors, including age, income, and financial independence.



Firstly, individuals under the age of 18 are typically considered minors and are often dependent on their parents or guardians for financial support. As a result, they may not have any significant income that requires them to file taxes, and their income might be included in their parent's or guardian's tax return. This leads to a lower percentage of filers in the under-18 group.


On the other hand, the 18-26 age group marks the transition into adulthood, where individuals begin to gain financial independence. Many start working full-time jobs or attend college, where they may earn income through part-time jobs or internships. This increased income leads to a higher percentage of filers in the 18-26 group, as they are now responsible for filing their own tax returns.


Furthermore, the age range of 18-26 also coincides with the period where individuals are more likely to have various income sources. This includes scholarships, grants, and student loans for those attending college. These additional income sources may also contribute to the increased percentage of filers in this age group.


Lastly, as individuals become more financially independent, they may become more aware of tax benefits and deductions available to them, such as educational credits or deductions for student loan interest. This newfound awareness could encourage more people within the 18-26 age group to file taxes, leading to a higher percentage of filers.


In conclusion, the dramatic jump in the percent of filers between the under-18 group and the 18-26 group can be attributed to factors such as increased financial independence, diverse income sources, and greater awareness of tax benefits and deductions.

To know more about filers refer here:

https://brainly.com/question/29614278#

#SPJ11

The number of views on a viral video can be modeled by the function
G(t) = 15(4)+3. Write an equivalent function of the form G'(t) = abt.

Answers

An equivalent function of the form G'(t) = abt for the given function G(t) = 15(4)+3 is: G'(t) = 63 * 1.

What is an equivalent function?

An equivalent function refers to a mathematical function that has the same output values or behavior as another function but may have a different mathematical expression or representation. In other words, two functions are considered equivalent if they produce the same results for the same inputs, even though they may be expressed differently in terms of mathematical notation, variables, or parameters.

According to the given information:

To write an equivalent function of the form G'(t) = abt, we need to rearrange the given function G(t) = 15(4)+3 into a format that matches the form G'(t) = abt.

The given function G(t) = 15(4)+3 can be simplified as follows:

G(t) = 60 + 3

G(t) = 63

Now, we can see that G(t) is a constant function with a constant value of 63. To express it in the form G'(t) = abt, we can rewrite it as:

G'(t) = 63 * 1

So, an equivalent function of the form G'(t) = abt for the given function G(t) = 15(4)+3 is:

G'(t) = 63 * 1

To know more about equivalent function visit: https://brainly.com/question/30196217

#SPJ1

Convers Corporation (calendar year-end) acquired the following assets during the current tax year: (ignore §179 expense and bonus depreciation for this problem): (Use MACRS Table 1, Table 2 and Table 5. )



Asset Date Placed in Service Original Basis



Machinery October 25 $ 92,000



Computer equipment February 3 32,000



Delivery truck* March 17 45,000



Furniture April 22 172,000



Total $ 341,000



*The delivery truck is not a luxury automobile. In addition to these assets, Convers installed new flooring (qualified improvement property) to its office building on May 12 at a cost of $520,000. A. What is the allowable MACRS depreciation on Convers’s property in the current year assuming Convers does not elect §179 expense and elects out of bonus depreciation? (Round your intermediate calculations and final answer to the nearest whole dollar amount. )

Answers

The allowable MACRS depreciation on Convers’s property in the current year assuming Convers does not elect §179 expense and elects out of bonus depreciation is $39,805.

To calculate the allowable MACRS depreciation, we need to determine the depreciation for each asset using the MACRS tables:

Machinery:

Placed in service in October, which is in the fourth quarter

Depreciation method: 5-year property

Percentage from Table 1: 20.00%

Basis for depreciation: $92,000

Depreciation for the current year: 20.00% x $92,000 x 0.5 = $9,200

Computer equipment:

Placed in service in February, which is in the first quarter

Depreciation method: 5-year property

Percentage from Table 1: 20.00%

Basis for depreciation: $32,000

Depreciation for the current year: 20.00% x $32,000 x 0.75 = $4,800

Delivery truck:

Placed in service in March, which is in the first quarter

Depreciation method: 5-year property

Percentage from Table 1: 20.00%

Basis for depreciation: $45,000

Depreciation for the current year: 20.00% x $45,000 x 0.75 = $6,750

Furniture:

Placed in service in April, which is in the second quarter

Depreciation method: 7-year property

Percentage from Table 2: 14.29%

Basis for depreciation: $172,000

Depreciation for the current year: 14.29% x $172,000 x 0.5 = $12,285

Flooring:

Placed in service in May, which is in the second quarter

Depreciation method: 39-year property

Percentage from Table 5: 2.564%

Basis for depreciation: $520,000

Depreciation for the current year: 2.564% x $520,000 x 0.5 = $6,670

The total allowable MACRS depreciation for Convers Corporation in the current year is the sum of the depreciation for each asset:

$9,200 + $4,800 + $6,750 + $12,285 + $6,670 = $39,805

Therefore, the allowable MACRS depreciation on Convers’s property in the current year is $39,805.

To know more on depreciation visit:

brainly.com/question/30531944

#SPJ11

Jerome wants to buy grass seed to cover his whole lawn, except for the pool. the pool is 4 5/6 m by 2 1/3 m. find the area the grass seed needs to cover.

Answers

Jerome will need approximately 138.72 square meters of grass seed to cover his lawn.

To find the area that the grass seed needs to cover, we first need to find the area of the entire lawn. Let's assume that the lawn is rectangular in shape.

Jerome hasn't given us the dimensions of the lawn, so let's say that it measures 10 meters by 15 meters. Therefore, the area of the entire lawn would be:

10 meters x 15 meters = 150 square meters

Now we need to subtract the area of the pool from the area of the entire lawn.

The pool measures 4 5/6 meters by 2 1/3 meters, which we can convert to improper fractions:

4 5/6 = (4 x 6 + 5) / 6 = 29 / 6
2 1/3 = (2 x 3 + 1) / 3 = 7 / 3

So the area of the pool would be:

29/6 meters x 7/3 meters = 203/18 square meters

To find the area that the grass seed needs to cover, we subtract the area of the pool from the area of the entire lawn:

150 square meters - 203/18 square meters

To subtract these two values, we need to get a common denominator:

150 square meters = (150 x 18) / 18 = 2700/18 square meters
203/18 square meters = 203/18 square meters

So the area that the grass seed needs to cover would be:

2700/18 square meters - 203/18 square meters = 2497/18 square meters

We can simplify this fraction by dividing the numerator and denominator by their greatest common factor, which is 1:

2497/18 square meters ≈ 138.72 square meters

Therefore, Jerome will need approximately 138.72 square meters of grass seed to cover his lawn, excluding the area around the pool.

To learn more about fraction, refer below:

https://brainly.com/question/10354322

#SPJ11

17 Question Evaluate y=−4(5)x y = − 4 ( 5 ) x for x

Answers

The equation y=−4(5)x y = − 4 ( 5 ) x can be simplified to y = -20x. This equation is a linear function with a slope of -20. As x increases, y decreases at a rate of 20 units for every one unit increase in x. Therefore, the value of y will decrease rapidly as x increases.

Circle A is located at (6, 5) and has a radius of 4 units. What is the equation of a line that is tangent to circle A from point C (2, 8)? x = 2 y = −0. 75x + 9. 5 y = 1. 33x + 1. 66 x = 8

Answers

We can use the point-slope form of the equation of a line to find the equation of the tangent line.

How to find the equation of the line that is tangent to circle A from point C (2, 8)?

To find the equation of the line that is tangent to circle A from point C (2, 8), we need to first find the point of tangency, which is the point where the line intersects the circle.

Point C (2, 8) is outside the circle, so the tangent line will be perpendicular to the line connecting the center of the circle to point C and will pass through point C.

Step 1: Find the center of the circle

The center of the circle A is at (6, 5).

Step 2: Find the slope of the line connecting the center of the circle to point C

The slope of the line connecting the center of the circle (6, 5) and point C (2, 8) is:

m = (8 - 5) / (2 - 6) = -3/4

Step 3: Find the equation of the line perpendicular to the line from Step 2 passing through point C

The slope of the line perpendicular to the line from step 2 is the negative reciprocal of the slope:

m_perp = -1 / (-3/4) = 4/3

Now we can use the point-slope form of the equation of a line to find the equation of the tangent line:

y - 8 = (4/3)(x - 2)

Simplifying, we get:

y = (4/3)x + 4.67

So the equation of the line that is tangent to circle A from point C (2, 8) is y = (4/3)x + 4.67.

Learn more about point-slope

brainly.com/question/837699

#SPJ11

I need someone to do this for me rq

Answers

Answer:

Step-by-step explanation:

The triangle area= 1/2 * the perpendicular height * breath

                           = 1/2*2*(3/4)

                          =0.75

How many grams of magnesium sulfate would be produced from the following reaction if 176 kJ of energy is absorbed by the reaction.


Al2(SO4)3 + 3 MgI2 → 2 AlI3 + 3 Mg(SO4) ΔH = +722 kJ

Answers

The amount of magnesium sulfate produced from the given reaction cannot be determined solely from the energy absorbed by the reaction.

Can the amount of magnesium sulfate produced from the reaction?

The given reaction is: Al2(SO4)3 + 3 MgI2 → 2 AlI3 + 3 Mg(SO4) and the enthalpy change for the reaction is ΔH = +722 kJ. The enthalpy change indicates that the reaction is endothermic, meaning that energy is absorbed by the reaction. The question asks about the amount of magnesium sulfate produced, which is not directly related to the energy absorbed by the reaction.

The balanced chemical equation shows that the reaction produces 3 moles of magnesium sulfate for every 3 moles of magnesium iodide consumed. Therefore, the amount of magnesium sulfate produced depends on the amount of magnesium iodide consumed.

To determine the amount of magnesium sulfate produced, we would need information about the amount of magnesium iodide consumed. The energy absorbed by the reaction does not provide enough information to determine the amount of magnesium sulfate produced.

In summary, the amount of magnesium sulfate produced from the given reaction cannot be determined solely from the energy absorbed by the reaction.

Learn more about Enthalpy

brainly.com/question/16720480

#SPJ11

Sophie deposited money into an account in which interest is compounded semiannually at a rate of 3.3%. She made no other deposits or withdrawals and the total amount in her account after 11 years was $19,786.19. How much did she deposit? Round answer to nearest whole number. Do not include units in the answer. Be sure to attach your work for credit.

Answers

Applying the compound interest formula, rounding to the nearest whole number, we get that Sophie deposited approximately $11,200.

How to Apply the Compound Interest Formula to Find How Much was Deposited?

We can use the formula for compound interest to solve this problem:

A = P * (1 + r/n)^(nt)

where A is the ending balance, P is the principal (the amount Sophie deposited), r is the annual interest rate (3.3%), n is the number of times the interest is compounded per year (2 for semiannual), and t is the number of years.

Substituting the given values, we get:

19786.19 = P * (1 + 0.033/2)^(2*11)

Simplifying and solving for P, we get:

P = 19786.19 / (1 + 0.033/2)^(2*11)

P ≈ 11200

Learn more about compound interest on:

https://brainly.com/question/30364118

#SPJ1

identify the inequalities for which the ordered pair (-1,-9) is a solution. Option C is y> -5/4x-3

Answers

The inequalities for which the ordered pair (-1,-9) is a solution are a and b

Identifying the ordered pairs of the inequality expression

From the question, we have the following parameters that can be used in our computation:

The inequality expression y> -5/4x-3 and the list of options

To determine the ordered pairs of the inequality expression, we set x = -1 and then calculate the value of y

Using the above as a guide, we have the following:

y > -5/4(-1) -3

Evauate

y > -1.75 -- this is false because -9 < -1.75


For the list of options, we have

Graph (a) True

Graph (b) True

Hence, the inequalities for which the ordered pair (-1,-9) is a solution are a and b

Read more about inequality at

https://brainly.com/question/25275758

#SPJ1

PLEASE HELP!!!!!!! Two lines, E and F, are represented by the equations given below. Line E: 5x + 5y = 40 Line F: x + y = 8 Which statement is true about the solution to the set of equations? (4 points) Question 2 options: 1) It is (40, 8). 2) It is (8, 40). 3) There is no solution. 4) There are infinitely many solutions.

Answers

Answer: (4)

Step-by-step explanation:

Two lines E and F are same.

5x + 5y = 40

x + y = 8

Deviding both hands of E by 5,

we get F's equation.

So every single point on the line x+y=8

represents the solution of the given system.

You get a job as a nurse. Your salary for the first year is $74,000. You will
receive a 1.2% increase every year. If you could save your entire salary, how
much money would you have in 5 years? Round to the nearest cent (2 decimal
places). Hint: What is a₁? What is r? Then use the formula for a finite
geometric series.

Answers

Answer: The amount of money you would have in 5 years if you could save your entire salary with a 1.2% increase every year would be $87,357.41.

Explanation:

The initial term, a₁, is $74,000, and the common ratio, r, is 1 + 1.2% = 1.012. To find the sum of the first 5 terms, we use the formula for a finite geometric series:

S₅ = a₁(1 - r⁶)/(1 - r)

Plugging in the values, we get:

S₅ = $74,000(1 - 1.012⁵)/(1 - 1.012) = $87,357.41 (rounded to the nearest cent)

Therefore, if you save your entire salary, you would have approximately $87,357.41 in 5 years with a 1.2% increase every year.

A ball is dropped from a window at a height of 36 feet. the function h(x) = -16x2 + 36 represents the height (in feet) of the ball after x seconds. round
to the nearest tenth.
how long does it take for the ball to hit the ground?

Answers

It takes about 1.5 seconds for the ball to hit the ground.

How to calculate the time for ball to hit the ground?

To find how long it takes for the ball to hit the ground, we need to find the value of x when h(x) = 0, since the height of the ball is 0 when it hits the ground. We can set -16x²+36 = 0 and solve for x:

-16x²+ 36 = 0

Dividing both sides by -16:

x² - 2.25 = 0

Adding 2.25 to both sides:

x²= 2.25

Taking the square root of both sides (we can ignore the negative root since time cannot be negative):

x = √(2.25) ≈ 1.5

Therefore, it takes about 1.5 seconds for the ball to hit the ground.

Learn more about ball

brainly.com/question/31068718

#SPJ11

The unit cube is divided into identical rectangular prisms. What is the volume of one of the identical prisms?

Answers

Note that the Volume of one prism = (1/16) unit³

How did we reach the above conclusion?

The given volume of the cube, v₁ = 1 unit cube

The height of the unit cube, h₁ = 1 unit

The length of the unit cube, w₁ = 1 unit

The height of the unit cube, l₁ = 1 unit

The number of identical prism located along the height = 2 identical prism

The number of identical prism located along the width = 2 identical prism

The number of identical prisms located along the length = 4 identical prism

Therefore;

The height of each identical rectangular prism that make up the unit cube, h₂ = h₁/2 = 1/2 unit

Similarly, for each identical rectangular prism, we have;

The width, w₂ = w₁/2 = 1/2 unit

The length, l₂ = l₁/4 = 1/4 unit

The volume of each one of the identical rectangular prism, v₂ = h₂ × w₂ × l₂



⇒  v₂ = 1/2 unit × 1/2 unit × 1/4 unit = 1/16 unit³

So it is correct to state that the volume of one of the identical rectangular prisms = (1/16) unit³.

Learn more about volume at:

https://brainly.com/question/1578538

#SPJ1

Full Question:

Although part of your question is missing, you might be referring to this full question:

See attached image.

The function
models the DVD sales, in billions of dollars, from 1999 to 2017, where
is the number of years since 1999.

What is the average rate of change of DVD sales, in billion of dollars, per year for the period from 2001 to 2008? Round your answer to the nearest tenth, if necessary.

Answers

The average rate of change of DVD sales is 723.1 billion dollars.

How to determine average rate of change of DVD sales?

The average rate of change of an equation can be determine by using the formula below:

average rate of change = (f(x₂)- f(x₁)) / [x₂- x₁]

From the given information of the DVD sales:

f(x) = -158.8x² + 2469.9x + 3010.5

x₁ = 2001 - 1999 = 2

x₂ = 2008 - 1999 = 9

average rate of change =  ([-158.8(9)² + 2469.9(9) + 3010.5] - [-158.8(2)² + 2469.9(2) + 3010.5]) / (9-2)

                                      = (12376.8 - 7315.1)/(7)

                                      = 723.1 billion dollars

Learn more about average rate of change on:

brainly.com/question/12363445

#SPJ1

Complete Question

Check image attached

Se estima que el costoanual de manejar cierto auto nuevo está dado por la fórmula C= 0. 35m + 2200, donde m representa el número de millas recorridas por año y C es el costo en dólares. Juana compró ese auto y decide presupuestar entre $6400 y $7100 para costos de manejo del año siguiente. ¿Cuál es el intervalo correspondiente de millas que ella puede manejar su nuevo auto? *

Answers

El intervalo correspondiente de millas que Juana puede manejar su nuevo auto está entre 10,000 y 14,000 millas.

How many miles can Juana drive her new car within the given budget range?

Para determinar el intervalo correspondiente de millas que Juana puede manejar su nuevo auto, podemos resolver la fórmula del costo anual en términos de la variable m (millas recorridas por año). Dado que el costo anual está entre $6400 y $7100, podemos establecer la siguiente desigualdad:

6400 ≤ 0.35m + 2200 ≤ 7100

Restando 2200 en los tres lados de la desigualdad, obtenemos:

4200 ≤ 0.35m ≤ 4900

Dividiendo por 0.35 en los tres lados, obtenemos:

12000 ≤ m ≤ 14000

Por lo tanto, Juana puede manejar su nuevo auto en un intervalo de millas que va desde 12000 millas hasta 14000 millas por año.

Learn more about correspondiente

brainly.com/question/24284278

#SPJ11

It takes a ship three hours to sail 72km with the current and 4 hours against it. Find the speed of the ship in still water and find the speed of the current

Answers

Answer:

The speed of the ship =21 km/h

Speed of the current = 3 km/h

Step-by-step explanation:

Let's denote the speed of the ship in still water as s and the speed of the current as c.

When the ship is traveling with the current, the effective speed is (s + c). (we have to sum up both the speeds) Therefore, in 3 hours, the ship can travel a distance of:

distance = speed × time = (s + c) × 3

We know that this distance is 72 km, so we can write:

(s + c) × 3 = 72

Simplifying this equation, we get:

s + c = 24

Similarly, when the ship is traveling against the current, the effective speed is (s - c). (The difference between the speeds). Therefore, in 4 hours, the ship can travel a distance of:

distance = speed × time = (s - c) × 4

We know that this distance is also 72 km, so we can write:

(s - c) × 4 = 72

Simplifying this equation, we get:

s - c = 18

We now have two equations:

s + c = 24 ; s - c = 18

We can solve for s and c by adding these two equations:

2s = 42

Therefore, s = 21 km/h.

Substituting this value of s into one of the equations above, we can solve for c:

s + c = 24

21 + c = 24

c = 3 km/h.

Therefore, the speed of the ship in still water is 21 km/h, and the speed of the current is 3 km/hs

Expert verification link :

https://brainly.com/question/9993302

Help Asap due Tomorrow in morning. Thanks if you help!

Answers

Answer: C. 54cm^2

Step-by-step explanation:

Break the shape into 2 pieces and multiply the sides. ex. 12 by 2 and 8 by 5 and then add the 2 answers to get 54cm

what is the unit rate of y=8x

Answers

The unit rate of the linear equation y = 8x is 8.

What is the unit rate of a linear equation?

A linear equation of a function illustrates the straight line on a coordinate plane. It can be expressed in a slope intercept form y = mx + b, where:

m = slopeb = y-intercept

The slope shows steep the gradient is and it is the change in the rise over the run. For a unit rate which is usually the ratio between two different quantities. We can say in the linear equation, the unit rate represents the slope of the function.

Given that:
y = 8x

To find the unit rate(slope) which is the change in the y-axis(rise) over the change in the x-axis(run) by using slope-intercept form. Then, we can conclude that the unit rate is 8.

Learn more about the unit rate of a linear equation here:

https://brainly.com/question/29337476

#SPJ1

Other Questions
Small baskets of tomatoes are sold at a vegetable stand for $3 per basket. Largebaskets of tomatoes are sold at the stand for $5 per basket. Only whole numbers ofbaskets may be purchased. A customer purchases a total of 8 baskets of tomatoes and pays $36. A. Write and solve a system of equations that models the number of smallbaskets (x) and the number of large baskets () that the customer purchases. Show or explain all your work. The Bantu make up nearly two-thirds of Africes population. Where is thisgroup located? Remember those walls I built?Well, baby they're tumbling downAnd they didn't even put up a fightThey didn't even make a soundI found a way to let you inBut I never really had a doubtStanding in the light of your haloI got my angel nowIt's like I've been awakenedEvery rule I had you breakin'It's the risk that I'm takingI ain't never gonna shut you out!Everywhere I'm looking nowI'm surrounded by your embraceBaby, I can see your haloYou know you're my saving graceYou're everything I need and moreIt's written all over your faceBaby, I can feel your haloPray it won't fade awayI can feel your halo, halo, haloI can see your halo, halo, haloI can feel your halo, halo, haloI can see your halo, halo. Halo, ooh ooh. Hit me like a ray of sunBurning through my darkest nightYou're the only one that I wantThink I'm addicted to your lightI swore I'd never fall againBut this don't even feel like fallingGravity can't begin to pull me back to the ground againAnd it's like I've been awakenedEvery rule I had you breakin'It's the risk that I'm takingI'm never gonna shut you out!Everywhere I'm looking nowI'm surrounded by your embraceBaby, I can see your haloYou know you're my saving graceYou're everything I need and moreIt's written all over your faceBaby, I can feel your haloPray it won't fade awayI can feel your halo, halo, haloI can see your halo, halo, haloI can feel your halo, halo, haloI can see your halo, halo. Halo, ooh ooh. I can feel your halo, halo, haloI can see your halo, halo, haloI can feel your halo, halo, haloI can see your halo, halo. Halo, ooh ooh. Halo, ooh ooh. Halo, ooh ooh, oh. Everywhere I'm looking nowI'm surrounded by your embraceBaby, I can see your haloYou know you're my saving graceYou're everything I need and moreIt's written all over your faceBaby, I can feel your haloPray it won't fade awayI can feel your halo, halo, haloI can see your halo, halo, haloI can feel your halo, halo, haloI can see your halo, halo. Halo, ooh oh. I can feel your halo, halo, haloI can see your halo, halo, haloI can feel your halo, halo, haloI can see your halo, halo. Halo, ooh oh How does the idea of race, ethnicity, and nationality impact one another? Researchers are trying to use radiocarbon dating to determine when the wooden buildings at an archaeological site were constructed. They know that trees in the area would have been cut down for the construction, and that when the trees died, they would have a ratio of carbon-14 to carbon-12 of about one part per trillion (1 ppt). The half-life for the decay of carbon-14 is 5,730 years, and they discover the ratio of carbon-14 to carbon-12 to be about 0. 125 parts per trillion (ppt). What is the age of the buildings at the site? *716. 25 years5,730 years17,190 years0. 125 billion years Jason Electronics, Inc. , is an electronics manufacturer that creates circuits for various electronic products. Currently, the company has eight locations. It prides itself on its fast production process. Rick is the newly appointed head operations manager of the companyand he is hoping to contribute to the company in a meaningful way. Rick has decided that there are several aspects of the production process that need to be changed or improved. One such aspect is quality control. He also wants to do some research to decide whether the company is moving in the right direction. In addition, he wishes to learn what other successful firms focus on so he can implement this knowledge at JEI. Refer to Jason Electronics, Inc. The plant layout at Jason Electronics is mostly likely a a. Capacity layout. B. Circular layout. C. Process layout. D. Fixed position layout. E. Product layout Fig 11 shows a cell from an organism. (a) (i) In which type of living organism are cells similar to the one shown in Fig. I.I normally found? (ii) State two reasons for your choice of living organism in (i) Fig. 1.2(a) shows the same cell after it had been placed in solution A for ten minutes. The cell was then transferred to solution B and Fig. 1.2(b) shows how it appeared after a further ten minutes (b) Explain what is occurred to cause the cell to appear as it does in Fig. 12(a) (c) Compared with the water potential of the cell, what can be deduced about solutions A and B (d) What will be found in region C in Fig 1.20b) at the end of the experiment and explain your answer Logy S500. MS Pre-Test Active 9 7 8 5 6 1 3 4 2 There are three principles of test construction: standardization, reliability, and validity. Please select the best answer from the choices provided T F a predatory bird is chasing its prey at 15.3 m/s when it emits a 202 hz squawk. ifthe prey is moving away from the predator at 13 m/s, what frequency will it hear ifthe air temperature is 27 c? In the coordinate plane, the point X(1,4) is translated to the point X(0,6) . Under the same translation, the points Y(-1,2) and Z(-3,1) are translated to Y and Z , respectively. What are the coordinates of Y and Z ?Help!!!!!! URGENT What was hamiltons logic in releasing the ""reynolds pamphlet""? do you agree with his decision? why or why not? what led to the creation of the Bantustans in south Africa An organism that exhibits a head with sensory equipment and a brain probably also ________. Group of answer choices is segmented is bilaterally symmetrical is diploblastic has a coelom Need this fastA) -24 Solve lim 69-24 B) 4 a+2 2-a C) 24 D) - 4 You are brainstorming with your co-founders about your next-year strategy. You are still doing R&D for your product, and you estimate that with probability 80% your product will be a success and bring you $200,000 in revenue. With probability 20% your product will fail and you will not take it to the market. In order to take the product to the market you estimate that you will need to raise $100,000. The interest rates right now are really low at 4%. On the other hand, if you wait until after your R&D is over, the interest rates will be higher and you expect them to be at 10% with probability 40% or at 15% with probability 60%. Remember that banks will always require the whole amount plus interest back regardless of whether your new venture fails or not. You can assume that if you get a loan, you will borrow the $100,000 amount for a whole year regardless of when you decide to take it. Should you get a loan now, before you see the results of the R&D, or should you wait for the results and then get funding depending on the outcome? What is your optimal strategy that maximizes your profit?(a) Draw the decision tree that helps with finding your optimal strategy. To get full score please provide the diagram and write down the probabilities.(b) What is your optimal strategy?(c) What is the smallest success probability (instead of the 80%) that would make getting the loan before you see the results of R&D more profitable? Hallar la altura de una asta bandera, si un estudiante la observa desde un punto a, con un ngulo de 30 y entre el estudiante y la asta hay una distancia de 10m. 16. Justin is joining a gym. The gym is currently offering a discount on the fee to join and on the monthly rate. The discounted price,in dollars,the gym charges can be represented by the equation y=10x+5 a. What are the slope and the Y-intercept of the equation? What do the slope and the Y-intercept each represent in this equation? Why is the Filipino language hated? A tank in the shape of a hemisphere has a diameter of 8 feet. If the liquid that fills the tank has a density of 86 pounds per cubic foot, what is the total weight of the liquid in the tank, to the nearest full pound? Plan a days menu (including snacks) for Olivia that contains at least 150 grams of carbohydrates without any refined sucrose (table sugar). Use the following average amounts of carbohydrates found in: